Uniform distribution and standard deviation

Click For Summary
The discussion revolves around calculating the standard deviation for a uniform distribution using specific integrals. The user derives expressions for the means m1 and m2 based on the uniform distribution parameters a and b. They express confusion over why a and b are assigned the values 0 and 1 to represent a standard distribution, despite the problem not explicitly stating this. Corrections to the algebra are suggested, emphasizing the importance of accurate calculations. Ultimately, the user resolves their issues with the algebra and confirms they arrive at the correct answer.
r0bHadz
Messages
194
Reaction score
17
Homework Statement
4.19:
According to one of the Western Electric rules for quality control, a produced item is considered conforming if its measurement falls within three standard deviations from the target value. Suppose that the process is in control so that the expected value of each measurement equals the target value. What percent of items will be considered conforming, if the distribution of measurements is: Uniform(a,b)?

4.20:
Refer to exercise 4.19, what percent of items falls beyond 1.5 standard deviations from the mean, if the distribution of measurements is Uniform(a,b)?
Relevant Equations
density f(x) = 1/(b-a), a<x<b
μ = (a+b)/2
σ = (b-a)/sqrt(12)
+(3/2) standard deviations from the mean = \frac {a+b}{12} + \frac{\sqrt3}{4} (b-a)
-(3/2) standard deviations from the mean = \frac {a+b}{12} - \frac{\sqrt3}{4} (b-a)

\frac {1}{b-a} \int_a^{\frac {a+b}{12} - \frac{\sqrt3}{4} (b-a)} dx = m_1= \frac {(-11+3\sqrt3)a + (1-3\sqrt3)b}{12(b-a)}

\frac {1}{b-a} \int_{\frac {a+b}{12} + \frac{\sqrt3}{4} (b-a)}^{b} dx = m_2 = \frac {(11-3\sqrt3)b + (-1+3\sqrt3)a}{12(b-a)}

adding m_1+m_2 I get:

\frac{(-12+6\sqrt3)a + (12-6\sqrt3)b}{12(b-a)}

I got up to this point where I quit the problem.

Now if I let b = 1 and a = 0 I get the answer to my problem which is .13397

I understand that this is the standard uniform distribution. But I don't understand what part of the problem statement let's me assign these values to a and b. Nothing in the problem statement tells me that this is not a general distribution, so why does it take values a = 0 and b = 1 to make it standard?
 
Physics news on Phys.org
You can always rescale to a standard distribution using a linear transformation.
 
  • Like
Likes FactChecker
r0bHadz said:
+(3/2) standard deviations from the mean = \frac {a+b}{12} + \frac{\sqrt3}{4} (b-a)
-(3/2) standard deviations from the mean = \frac {a+b}{12} - \frac{\sqrt3}{4} (b-a)

You shouldn't have ##12## in those two means above, or below.

\frac {1}{b-a} \int_a^{\frac {a+b}{12} - \frac{\sqrt3}{4} (b-a)} dx = m_1= \frac {(-11+3\sqrt3)a + (1-3\sqrt3)b}{12(b-a)}

\frac {1}{b-a} \int_{\frac {a+b}{12} + \frac{\sqrt3}{4} (b-a)}^{b} dx = m_2 = \frac {(11-3\sqrt3)b + (-1+3\sqrt3)a}{12(b-a)}

adding m_1+m_2 I get:

\frac{(-12+6\sqrt3)a + (12-6\sqrt3)b}{12(b-a)}

Factor a minus out of the first term and simplify it.

I got up to this point where I quit the problem.

Now if I let b = 1 and a = 0 I get the answer to my problem which is .13397
Fix the algebra and you will get the same answer.
 
LCKurtz said:
Fix the algebra and you will get the same answer.

ended up fixing it, and got the same answer >.> damn I hate algebra
 
Question: A clock's minute hand has length 4 and its hour hand has length 3. What is the distance between the tips at the moment when it is increasing most rapidly?(Putnam Exam Question) Answer: Making assumption that both the hands moves at constant angular velocities, the answer is ## \sqrt{7} .## But don't you think this assumption is somewhat doubtful and wrong?

Similar threads

Replies
3
Views
2K
  • · Replies 8 ·
Replies
8
Views
2K
  • · Replies 10 ·
Replies
10
Views
1K
  • · Replies 6 ·
Replies
6
Views
2K
  • · Replies 9 ·
Replies
9
Views
2K
  • · Replies 3 ·
Replies
3
Views
3K
Replies
6
Views
1K
Replies
9
Views
4K
  • · Replies 4 ·
Replies
4
Views
3K
  • · Replies 2 ·
Replies
2
Views
2K